the sum of 3 times a number and 7 is 8 use the variable of w for the unknown number

Answers

Answer 1

Answer:

Step-by-step explanation:

I solved it in the picture above

The Sum Of 3 Times A Number And 7 Is 8 Use The Variable Of W For The Unknown Number
Answer 2

Answer:

w = [tex]\frac{1}{3}[/tex]

Step-by-step explanation:

The equation representing the situation is

3w + 7 = 8 ( subtract 7 from both sides )

w = 1 ( divide both sides by 3 )

w = [tex]\frac{1}{3}[/tex]


Related Questions

seventy four lakh nine thousand eleven in numeral form​

Answers

Seventy Four Lakh , Nine Thousand And Eleven=

7409011

The probability distribution for a random variable x is given in the table X: -5,-3,-2,0,2,3 Probability: .17,.13,.33,.16,.11,.10 Find the probability that X <_-3

Answers

Answer:

0.3 probability that [tex]x \leq -3[/tex]

Step-by-step explanation:

The probability distribution is given in the table.

Probability that x <= -3

The values that are -3 or lower are -3 and -5. So

[tex]P(X \leq -3) = P(X = -3) + P(X = -5)[/tex]

From the table:

[tex]P(X = -3) = 0.13, P(X = -5) = 0.17[/tex]. So

[tex]P(X \leq -3) = P(X = -3) + P(X = -5) = 0.13 + 0.17 = 0.3[/tex]

0.3 probability that [tex]x \leq -3[/tex]

Answer:0.3

Step-by-step explanation:

xác định m để 3 vector sau đây phụ thuộc tuyến tính
u=(m+1,1,m+1),v=(1,1,1),u(2,0,m+2



0

Answers

Answer:

i dont know hahaha

Step-by-step explanation:

soryy

Convex angles help me

Answers

Answer:

C, D, F

Step-by-step explanation:

Shape A is not a polygon; it has a line that doesn't connect anywhere. Even if it is a polygon, it would be concave. Shape B is a concave polygon, shape E is also a concave polygon, shape G and H are also concave polygons. Only shapes C, D, and F are convex polygons. Concave polygons are shapes that cave in, and convex polygons are caves that don't cave in.

Can someone answer with steps and explanation? Thanks.

Answers

Answer:

[tex]x=-16\text{ or } x=7[/tex]

Step-by-step explanation:

Since ΔABC is mapped onto ΔDEF, we can write that:

[tex]\Delta ABC\cong \Delta DE F[/tex]

By CPCTC:

[tex]\angle A\cong \angle D[/tex]

And since ΔABC is isosceles with Vertex C:

[tex]\angle A \cong \angle B[/tex]

We are given that:

[tex]m\angle D=34[/tex]

Hence:

[tex]m\angle A=34=m\angle B[/tex]

We are also given that:

[tex]m\angle C=x^2+9x[/tex]

The interior angles of a triangle must sum to 180°. Thus:

[tex]m\angle A+m\angle B+m\angle C=180[/tex]

Substitute:

[tex](34)+(34)+(x^2+9x)=180[/tex]

Simplify:

[tex]68+x^2+9x=180[/tex]

Isolate the equation:

[tex]x^2+9x-112=0[/tex]

Factor:

[tex](x+16)(x-7)=0[/tex]

Zero Product Property:

[tex]x+16=0\text{ or } x-7=0[/tex]

Solve for each case:

[tex]x=-16\text{ or } x=7[/tex]

Testing the solutions, we can see that both yields C = 112°.

Hence, our solutions are:

[tex]x=-16\text{ or } x=7[/tex]

Stats question what are the main difference between frequenting and Bayesian

Answers

Answer:

The frequentist believes that probability represents long term frequencies of repeatable events (such a flipping a coin). Frequentists do not attach probabilities to hypotheses or unknown values. On the other hand, the Bayesian approach uses probabilities to represent the uncertainty in any event or hypothesis.

HOPE IT HELPS

What is the value of the expression 3m-4.2 If m equals 2.1

Answers

Answer:

2.1

Step-by-step explanation:

Given :

3m-4.2 where, m=2.1

Now,

3(2.1)-4.2

6.3-4.2

2.1

Answer is 2.1

How many liters each of a 25% acid solution and a 50% acid solution must be used to produce 80 liters of a 40% acid solution?

Answers

Answer:

32 and 48 liters

Step-by-step explanation:

Let 25% solution is x liters, then 50% solution is (80 - x) liters.

Acid content is going to be same:

0.25x + 0.5(80 - x) = 80*0.40.25x - 0.5x + 40 = 320.25x = 8x = 8/0.25x = 32 liters

So 32 liters of 25% solution and 80 - 32 = 48 liters of 50% solution

Simplify the expression.

33 · 32 + 12 ÷ 4

Answers

Answer:

1059

Step-by-step explanation:

33 · 32 + 12 ÷ 4

PEMDAS

Multiply and divide first from left to right

1056 + 3

Then add

1059

[tex]\huge\textsf{Hey there!}[/tex]

[tex]\mathsf{33\times32+12\div4}\\\\\mathsf{33\times32= \boxed{\bf 1,056}}\\\\\mathsf{\bold{1,056}+12\div4}\\\\\mathsf{12\div4=\boxed{\bf 3}}\\\\\mathsf{1,056+\bf 3}\\\mathsf{= \boxed{\bf 1,059}}\\\\\\\boxed{\boxed{\large\textsf{Answer: \huge \bf 1,059}}}\huge\checkmark[/tex]

[tex]\large\textsf{Good luck on your assignment and enjoy your day!}[/tex]

~[tex]\frak{Amphitrite40:)}[/tex]

andrea is planning a birthday party and wants to include a cheese board with the desserts.
she reads online that she should have 110g of cheese per person ,but the cheese is sold in blocks of 500g
How many blocks of cheese should she buy to ensure that each guest can have 110g of cheese?​

Answers

Step-by-step explanation:

how many people in the party please ?

I need help k please help gardinuhola

Answers

Answer:

Step-by-step explanation:

for circle

diameter = 26 cm

radius = diamtere/2

=26/2

=13 cm

area of circle  = πr^2

=3.14^13^2

=3.14*169

=530.66 cm^2

=531 cm^2   (after round off )

area of square= l^2

=5.1^2

=26.01 mi^2

are of rectangle = l *b

=6*5.1

=30.6 m^2

area of triangle = base*height / 2

=9*6.4 / 2

=57.6 / 2

=28.8 yd^2

he speeds (in MPH) of automobiles traveling in a city are given below:
20, 35, 42, 52, 65, 49, 24, 37, 23, 41, 50, 58
The mean speed of the cars is

Answers

Answer:

Mean speed = 41.3 mph

Step-by-step explanation:

Given that,

The speeds of an automobiles are given below:

20, 35, 42, 52, 65, 49, 24, 37, 23, 41, 50, 58

We need to find the mean speed of the cars.

Mean = sum of observations/ no. of observation

[tex]M=\dfrac{20+35+42+52+65+49+24+37+23+41+50+58}{12}\\\\M=41.3[/tex]

So, the mean speed of the cars is equal to 41.3 mph.

solve marked question only !
plz

Answers

Answer:

hello ok I will share u the link where u can find step by step answers

Answer:

The angle of elevation of the top of a tower from a point on the ground, which is 30 m away from the foot of the tower, is 30°. Find the height of the tower.

A Let tower be AB

Let point be C

Distance of point C from foot of tower = 30m Hence,

BC = 30m

Angle of elevation = 30°

So < ACB = 30°

Since tower is vertical,

< ABC = 90°

Is 9/54 equivalent to 1/3

Answers

Answer: No.

Step-by-step explanation:

9/54 is equal to 1/6

Answer:

no, it is not

Step-by-step explanation:

9/54 can be simplified by dividing both sides by 3, getting 3/18. you can do this again and get 1/6. significantly smaller that l/3

A person walks 1/6 mile in 1/18 hour.

The person's speed is _ miles per hour.

Answers

This Is What I Got!

Hope This Helps! :)

Have A Good Day!!

And If You Can I Wouldn't Mind A Brainliest! :))

Answer:

Divide 1/6 miles to 1/12hour since u wanna find our miles per hour

So it’ll be : 1/6 / 1/12

= 1/6 x 12/1

= 2 miles

Polygon ABCDE is reflected to produce polygon A’ B’ C’ D’ E’. What is the equation for the line of reflection?

Options:
A. y=0
B. x=0
C. x=2
D. y=1

Answers

Answer:

B. x=0

Step-by-step explanation:

The Polygon ABCDE is reflected by

y-axis => x=0 , to produce polygon A’ B’ C’ D’ E’

Answer:

B. x = 0

Step-by-step explanation:

Plato

What is the average score of runa 140,96 and 13?​

Answers

[tex]\huge\textsf{Hey there!}[/tex]

[tex]\large\textsf{Formula:}[/tex]

[tex]\mathsf{\dfrac{Sum\ of\ all\ terms}{Total\ number\ of\ values\ in\ the\ data}= your\ mean/average}[/tex]

[tex]\mathsf{\dfrac{140+96+13}{3}}[/tex]

[tex]\mathsf{140+36+13}[/tex]

[tex]\mathsf{140 + 96 = \bf 236}[/tex]

[tex]\mathsf{236 + 13}[/tex]

[tex]\mathsf{ = \bf 249}[/tex]

[tex]\mathsf{\dfrac{249}{3}}[/tex]

[tex]\mathsf{= \bf 83}[/tex]

[tex]\boxed{\boxed{\large\textsf{Possible answer: \huge \bf 83}}}\huge\checkmark[/tex]

[tex]\large\textsf{Good luck on your assignment and enjoy your your day!}[/tex]

~[tex]\frak{Amphitrite1040:)}[/tex]

2 divided by 0.75 full divison work i dont just need the answer​

Answers

Answer:

0.375

Step-by-step explanation:

Check the picture below.

whenever we do division of decimals, we have to mind how many decimals are there on each amount, the dividend as well as the divisor, that way we pad with zeros the other amount accordingly whilst losing the dot, for example, to say divide 3 by 0.123, 3 has no decimals, whilst 0.123 has three decimals, so we can just divide 3000 by 0123, so dividing 3 by 0.123 is the same as dividing 3000 by 123.  Another example, if we were to divide say 23.761 by 555.89331, the dividend has 3 decimals, that means 3 zeros the other way, the divisor has 5 decimals, that means 5 zeros the other way while losing the dots, so we'd end up dividing 2376100000 by 55589331000, which we can simplify to just 2376100 by 5589331, as you can see in the picture in this case.

Suppose that g(x)= f(x)+ 6. Which statement best compares the graph of g(x) with the graph of f(x)?

A. The graph of g(x) is the graph of f(x) shifted 6 units down.

B. The graph of g(x) is the graph of f(x) shifted to the right.

C. The graph of g(x) is the graph of f(x) shifted 6 units to the left.

D. The graph of g(x) is the graph of f(x) shifted 6 units up.

Answers

Answer:

D

Step-by-step explanation:

The + 6 moves it up 6 units.

The correct answer is (D) "The graph of g(x) is the graph of f(x) shifted 6 units up."

What is the function?

A relationship between a group of inputs and one output is referred to as a function. In plain English, a function is an association between inputs in which each input is connected to precisely one output. A domain, codomain, or range exists for every function. Typically, f(x), where x is the input, is used to represent a function.

When we add a constant to a function, such as in the case of g(x) = f(x) + 6, it will shift the graph of f(x) upward by 6 units.

This is because, for any value of x, the value of f(x) will be added to 6, resulting in a vertical shift of the entire graph.

Option (A) is incorrect because adding 6 to f(x) would shift the graph up, not down.

Option (B) is incorrect because adding a constant to a function does not cause it to shift horizontally.

Option (C) is incorrect because adding 6 to f(x) would shift the graph right, not left.

D. The graph of g(x) is the graph of f(x) shifted 6 units up. Adding a constant term to a function will shift the graph of the function vertically. In this case, adding 6 to f(x) will shift the graph of f(x) upward by 6 units, resulting in the graph of g(x).

Learn more about function here:

https://brainly.com/question/29633660

#SPJ7

what'd the greatest common factor (GCF) for each pair of numbers. 25, 55 The GCE IS​

Answers

Answer:

5

Step-by-step explanation:

5 l 25,55

   l 5,11

4(x+9)=8x-7
Solve for x

Answers

Answer:

x = 10.75

Step-by-step explanation:

4(x+9)=8x-7

Step 1: Distribute the 4 to the x and 9

4 * x = 4x

4 * 9 = 36

We now have 4x + 36 = 8x - 7

Step 2: add 7 to both sides

36 + 7 = 43

-7 + 7 cancels out

We now have 4x + 43 = 8x

Step 3 subtract 4x from both sides

4x - 4x cancels out

8x - 4x = 4x

We now have 4x = 43

step 4 divide both sides by 4

4x / 4 ( the 4s cancel out and we're left with x )

43/4 = 10.75

We're left with x = 10.75

I need help I’ll give u brainlest

Answers

Answer:

[tex]V=280[/tex] cubic inches

Step-by-step explanation:

Volume formula for triangular prism is  [tex]V=\frac{1}{2} bhl[/tex]

[tex]V=\frac{1}{2} (7)(10)(8)[/tex]

[tex]V=\frac{1}{2}(560)[/tex]

[tex]V=280[/tex]

Hope this helps

The top of the Boulder Dam has an angle of elevation of 1.2 radians from a point on the Colorado River. Measuring the angle of elevation to the top of the dam from a point 155 feet farther down river is 0.9 radi- ans; assume the two angle measurements are taken at the same elevation above sea level. How high is the dam?

Answers

Answer:

382.925 feets

Step-by-step explanation:

The solution diagram is attached below :

Converting radian measurement to degree :

radian angle * 180/π = degree angle

1.2 * 180/π = 68.755°

0.9 * 180/π = 51.566°

Height of dam is h:

Using trigonometry :

Tan θ = opposite / Adjacent

Tan 68.755° = h / x

h = x Tan 68.755° - - - (1)

Tan 51.566° = h / (155+x)

h = (155+x) tan 51.566° - - - (2)

Equate (1) and (2)

x Tan 68.755 = (155+x) Tan 51.566

x Tan 68.755 = 155tan 51.566 + x tan 51.566

x Tan 68.755 = 195.32311 + x Tan 51.566

x Tan 68.755 - x Tan 51.566 = 195.32311

x(tan 68.755 - tan 51.566) = 195.32311

x * 1.3120110 = 195.32311

1.3120110x = 195.32311

x = 195.32311 / 1.3120110

x = 148.87307

Using :

h = x Tan 68.755

h = 148.87307 * tan(68.755)

h = 382.92539

h = 382.925 feets

Bạn dự định một năm nữa kể từ hôm nay, bạn sẽ mở một tài khoản tiết kiệm 100 triệu đồng với lãi suất niêm yết của ngân hàng là 8%/ năm
a, Nếu bạn chia tài khoản tiền gửi 100 triệu đônbf ghành 5 tài khoản tiền bằng nhau, mỗi tài khoản 20 triệu đồng để gửi vào cuối năm 1,2,3,4 và 5 thì số dư tài khoản tiền gửi của bạn sẽ là bao nhiêu vào cuối năm thứ 5, với lãi suất 8%/năm và ghép lãi hằng năm ?

Answers

Answer:

Sorry dont understand

international system of 89643092 in words​

Answers

Answer:

Eighty nine million six hundred forty three thousand ninety two

Step-by-step explanation:

89,643,092

=> Eighty nine million six hundred forty three thousand ninety two

The diagram shows a right-angled triangle.
xo
26 cm
17 cm
Find the size of angle x.
Give your answer correct to 1 decimal place.

Answers

Answer:

Diagram? I don't see a diagram.

Where is the diagram?

Step-by-step explanation:

Calculate the area of the following Circle

Answers

Answer:

324

Step-by-step explanation:

Circles are 100% even. If you add another line going opposite direction you would multiply 18x18. You get 324.

Determine whether the following event is mutually exclusive or not mutually exclusive.

Choosing a student who is a mathematics major or a business major from a nearby university to participate in a research study. (Assume that each student only has one major.)

Answers

Answer:

The event is mutually exclusive.

Step-by-step explanation:

Mutually exclusive events are events that cannot exist simultaneously.

Thus, events that are not mutually exclusive can exist simultaneously.

Since each student only has one major, a single student cannot be both a mathematics major and a business major.

So, the event is mutually exclusive.

Darcy gave her hairstylist a $ 4.90 The tip was 14​% of the cost of the haircut . Write an equation to find​ b, the cost of the haircut.

Answers

Answer:

Equation: 4.90/b = 14/100

Solution: b = $35

Step-by-step explanation:

Variable b = cost of the haircut

Solve for b:

4.90/b = 14/100

490 = 14b

35 = b

Check your work:

35 × 0.14 = 4.90

Correct!

Who can answer this? I’ll mark brainliest!!

Answers

Answer:

y = 0.48(x - 0.5)² - 3

y = 0.48(x² - x - 6)

Step-by-step explanation:

From the graph the zeros are

x = {-2, 3}

The x coordinate of the vertex is the midpoint of the roots

x = (-2 + 3) / 2

x = 0.5

The y coordinate of the vertex is

y = -3

vertex = (0.5, -3)

--------------------------------------

Merhod I - vertex

Vertex form is

y = a(x - h)² + k

plug in the vertex

y = a(x - 0.5)² - 3

to find a plug in either root

using x = 3

0 = a(3 - 0.5)² - 3

0 = a(2.5)² - 3

0 = 6.25a - 3

3 = 6.25a

a = 3/6.25

a = 0.48

y = 0.48(x - 0.5)² - 3

-----------------------------

Method II - roots

y = a(x + 2)(x - 3)

-3 = a(0.5 + 2)(0.5 - 3)

-3 = a(2.5)(-2.5)

-3 = -6.25a

3/6.25 = a

0.48 = a

y = 0.48(x + 2)(x - 3)

Expand

y = 0.48(x² - x - 6)

Other Questions
Which three factors determine the formality of a discussion?O opinion, audience, and timeO topic, audience, and purposeO facts, purpose, and locationO topic, time, and location Write a loop that continually asks the user what food the user has in their refrigerator until the user enters apples, in which case the loop ends. It should acknowledge the user in the following format. For the first food, the user might say "hamburger," so your response would be, "You have a hamburger with a total of 1 food(s) if they enter hamburger, and so on until they enter apples at which point the loop ends. ---- make sure you save your file as "may28.py" * **PLEASE HELP**The frequency table below represents the 30 best battling averages for a semi pro baseball league. Which ranges of battling averages were least common among the players What is the perimeter of this rectangle? what is a reflection of a story X+ 5If m(x) =x-1 and n(x) = x-3, which function has the same domain as (mon)(x)?X+5O (x)=1111o h(x)=X-111O (X)=X-411Oh(x) =X-3 Does anybody know the answer to this? Who was the first person to ever make a famous painting. Which of these was not one of the main causes of the devastating explosion in Chernobyl?A. radiationB. excess steamC. human errorD. hydrogen A scientist adds 5 mcg of the mineral cadmium to the daily diet of 100 4-week-old laboratory mice. After twelve weeks, the researcher weighs and takes blood samples from each mouse. According to her findings, the mice lost weight during the 12-week period and they have abnormal levels of certain hormones in their blood. Based on this information and your knowledge of scientific research, what would you tell the researcher about her findings?A. She needs to conduct more tests on the animals' blood, because her findings areincomplete.B. The findings are not meaningful or valid because of the way she designed her study.C. Her results are very interesting and she should report her findings to nutrition scientists, sothey can repeat her study and confirm the results.D. The findings need to be summarized in a research article for submission to a peer-reviewednutrition journal. Neglecting air resistance and the weight of the propellant, determine the work done in propelling a five-ton satellite to a height of (a) 100 miles above Earth and (b) 300 miles above Earth. The frequency of an X ray is 6.5 x 1018 Hz. What is the X rays wavelength? . Prepare general journal entries to record the transactions below for Spade Company by using the following accounts: Cash; Accounts Receivable; Office Supplies; Office Equipment:; Accounts Payable; Recording effects of K. Spade, Capital: K. Spade, Withdrawals; Fees Earned; and Rent Expense. Use the letters beside each transactions in T-accounts transaction to identify entries. After recording the transactions, post them to T-accounts, which serves as A the general ledger for this assignment. Determine the ending balance of each T-account. a. Kacy Spade, owner, invested $100,750 cash in the company b. The company purchased office supplies for $1,250 cash. c. The company purchased S10,050 of office equipment on credit. d. The company received S15,500 cash as fees for services provided to a customer. e. The company paid $10,050 cash to settle the payable for the office equipment purchased in transaction c f. The company billed a customer $2,700 as fees for services provided. g. The company paid $1,225 cash for the monthly rent. h. The company collected S1,125 cash as partial payment for the account receivable created in transaction f. i. Kacy Spade withdrew $10.000 cash from the company for personal use. To solve the equation 6x + 3 = 9 for x, what operations must beperformed on both sides of the equation in order to isolate the variablex? Find an equation for the line with the given property. (a) It passes through the point (2, 6) and is parallel to the line 4x + y 10 = 0.It has x-intercept 6 and y-intercept 4. Suppose b is any integer. If b mod 12 = 7, what is 4b mod 12? In other words, if division of b by 12 gives a remainder of 7, what is the remainder when 4b is divided by 12? Fill in the blanks to show that the same answer will be obtained no matter what integer is used for b at the start. Because b mod 12 = 7, there is an integer m such that b = 12m + . Multiply both sides of this equation by 4 and then simplify the right-hand side to find values of q and r such that 4b = 12q + r with 0 r < 12. The result is q = and r = . Now 0 r < 12, and q is an integer because ---Select--- . So the uniqueness part of the quotient remainder theorem guarantees that the remainder obtained when 4b is divided by 12 is . Need Help? A bacteria culture is growing at a rate of r(t) = 7e^0.6t thousand bacteria per hour after t hours. How much did the bacteria population increase during the first two hours? (Round your answer to three decimal places.) An effort of 20N is applied to lift a load.If the load arm and effort arm are 15cm and 60cm respectively,calculate the load. 9. If you handwrite your draft, you should confirmpunctuationcapitalizationlegibilitygrammar is a collection of information stored under a single name what living things can be found in mangrove forest and tropical rainforest